Inner 6-7-8 triangle

Calculus Level 5

There exists a point inside a triangle such that it is 6, 7 and 8 units away from the triangle's vertices.

Let the side lengths of this triangle be denoted as a , b a,b and c c as shown above.

Among all the possible ways to construct such a triangle above, the largest possible perimeter can be obtained when a , b , c a,b,c satisfy the equation a 2 + P 2 a 2 + Q = b 2 + R 2 b 2 = c 2 + S 2 c 2 + T , a^2 + \dfrac{P^2}{a^2} + Q = b^2 +\dfrac{R^2}{b^2} = c^2 +\dfrac{S^2}{c^2} + T ,

where P , Q , R , S , T P,Q,R,S,T are constant positive integers. What is P + Q + R + S + T ? P+Q+R+S+T ?

Inspiration . I was trying to find the closed form of p p there but I went into a rabbit hole and formulated this question.


The answer is 138.

This section requires Javascript.
You are seeing this because something didn't load right. We suggest you, (a) try refreshing the page, (b) enabling javascript if it is disabled on your browser and, finally, (c) loading the non-javascript version of this page . We're sorry about the hassle.

3 solutions

Mark Hennings
May 17, 2021

If the central angles are α , β , γ \alpha,\beta,\gamma , then a = 85 84 cos α a = \sqrt{85 - 84\cos\alpha} , b = 113 112 cos β b = \sqrt{113-112\cos\beta} and c = 100 96 cos γ c = \sqrt{100-96\cos\gamma} , and so we want to maximise a + b + c = 85 84 cos α + 113 112 cos β + 100 96 cos γ a + b + c \; = \; \sqrt{85 - 84\cos\alpha} + \sqrt{113-112\cos\beta} + \sqrt{100-96\cos\gamma} subject to the restraint α + β + γ = π \alpha + \beta + \gamma = \pi . Lagrange multipliers tell us that 42 sin α a = 56 sin β b = 48 sin γ c = k \frac{42\sin\alpha}{a} \; = \; \frac{56\sin\beta}{b} \; = \; \frac{48\sin\gamma}{c} \; =\; k for some contant k k , and hence k 2 = 4 2 2 a 2 [ 1 ( 85 a 2 84 ) 2 ] = 5 6 2 b 2 [ 1 ( 113 b 2 112 ) 2 ] = 4 8 2 c 2 [ 1 ( 100 c 2 96 ) 2 ] = ( a 2 1 ) ( 169 a 2 ) 4 a 2 = ( b 2 1 ) ( 225 b 2 ) 4 b 2 = ( c 2 4 ) ( 196 c 2 ) 4 c 2 226 4 k 2 = a 2 + 1 3 2 a 2 + 56 = b 2 + 1 5 2 b 2 = c 2 + 2 8 2 c 2 + 26 \begin{aligned} k^2 & = \; \frac{42^2}{a^2}\left[1 - \left(\frac{85 - a^2}{84}\right)^2\right] \; = \; \frac{56^2}{b^2}\left[1 - \left(\frac{113-b^2}{112}\right)^2\right] \; = \; \frac{48^2}{c^2}\left[1 - \left(\frac{100 - c^2}{96}\right)^2\right] \\ & = \; \frac{(a^2-1)(169-a^2)}{4a^2} \; = \; \frac{(b^2-1)(225-b^2)}{4b^2} \; = \; \frac{(c^2-4)(196-c^2)}{4c^2} \\ 226 - 4k^2 & = \; a^2 + \frac{13^2}{a^2} + 56 \; = \; b^2 + \frac{15^2}{b^2} \; = \; c^2 + \frac{28^2}{c^2} + 26 \end{aligned} making the answer 13 + 56 + 15 + 28 + 26 = 138 13 + 56 + 15 + 28 + 26 = \boxed{138} .

Jon Haussmann
May 18, 2021

Let A A , B B , C C , P P be points in the plane such that x = A P x = AP , y = B P y = BP , z = C P z = CP are fixed distances. Then by the arguments in the following links, when the perimeter of triangle A B C ABC is maximized, P P is the incenter of triangle A B C ABC .

https://math.stackexchange.com/questions/1684501/finding-max-perimeter-of-triangle-of-three-circulating-points

https://www.cut-the-knot.org/Curriculum/Geometry/TripolarOptimization.shtml

Let r r be the inradius of triangle A B C ABC . Then r r is the height of triangle P B C PBC with respect to base B C BC , so by Heron's formula, r 2 = ( 2 [ P B C ] B C ) 2 = 4 [ P B C ] 2 a 2 = ( ( y + z ) 2 a 2 ) ( a 2 ( y z ) 2 ) 4 a 2 = a 4 + 2 ( y 2 + z 2 ) a 2 ( y 2 z 2 ) 2 4 a 2 . \begin{aligned} r^2 &= \left( \frac{2 [PBC]}{BC} \right)^2 \\ &= \frac{4 [PBC]^2}{a^2} \\ &= \frac{((y + z)^2 - a^2)(a^2 - (y - z)^2)}{4a^2} \\ &= \frac{-a^4 + 2(y^2 + z^2) a^2 - (y^2 - z^2)^2}{4a^2}. \end{aligned} Then 4 r 2 = a 2 2 ( y 2 + z 2 ) + ( y 2 z 2 ) 2 a 2 . -4r^2 = a^2 - 2(y^2 + z^2) + \frac{(y^2 - z^2)^2}{a^2}.

Likewise, 4 r 2 = b 2 2 ( x 2 + z 2 ) + ( x 2 z 2 ) 2 b 2 , 4 r 2 = c 2 2 ( x 2 + y 2 ) + ( x 2 y 2 ) 2 c 2 , \begin{aligned} -4r^2 &= b^2 - 2(x^2 + z^2) + \frac{(x^2 - z^2)^2}{b^2}, \\ -4r^2 &= c^2 - 2(x^2 + y^2) + \frac{(x^2 - y^2)^2}{c^2}, \end{aligned} so a 2 2 ( y 2 + z 2 ) + ( y 2 z 2 ) 2 a 2 = b 2 2 ( x 2 + z 2 ) + ( x 2 z 2 ) 2 b 2 = c 2 2 ( x 2 + y 2 ) + ( x 2 y 2 ) 2 c 2 . a^2 - 2(y^2 + z^2) + \frac{(y^2 - z^2)^2}{a^2} = b^2 - 2(x^2 + z^2) + \frac{(x^2 - z^2)^2}{b^2} = c^2 - 2(x^2 + y^2) + \frac{(x^2 - y^2)^2}{c^2}.

Plugging in x = 8 x = 8 , y = 6 y = 6 , z = 7 z = 7 gives us a 2 170 + 1 3 2 a 2 = b 2 226 + 1 5 2 b 2 = c 2 200 + 2 8 2 c 2 , a^2 - 170 + \frac{13^2}{a^2} = b^2 - 226 + \frac{15^2}{b^2} = c^2 - 200 + \frac{28^2}{c^2}, which leads to a 2 + 1 3 2 a 2 + 56 = b 2 + 1 5 2 b 2 = c 2 + 2 8 2 c 2 + 26. a^2 + \frac{13^2}{a^2} + 56 = b^2 + \frac{15^2}{b^2} = c^2 + \frac{28^2}{c^2} + 26.

Pi Han Goh
May 15, 2021

Let θ 1 , θ 2 \theta_1, \theta_2 and θ 3 \theta_3 denote the interior angles that split the original triangle into 3 triangles.

By cosine rule , cos θ 1 = 85 a 2 84 cos θ 2 = 113 b 2 112 cos θ 3 = 100 c 2 96 \cos\theta_1 = \frac{85-a^2}{84} \hspace{5em} \cos\theta_2 = \frac{113-b^2}{112} \hspace{5em} \cos\theta_3 = \frac{100-c^2}{96}

We want to maximize P : = a + b + c = 85 84 cos θ 1 + 113 112 cos θ 2 + 100 96 cos θ 3 P := a + b + c = \sqrt{85 - 84 \cos \theta_1} + \sqrt{113 - 112 \cos \theta_2} + \sqrt{100 - 96 \cos\theta_3}

Note that cos ( θ 3 ) = cos [ 2 π ( θ 1 + θ 2 ) ] = cos ( θ 1 + θ 2 ) = cos ( θ 1 ) cos ( θ 2 ) sin ( θ 1 ) sin ( θ 2 ) . \cos(\theta_3) = \cos[2\pi - (\theta_1 + \theta_2)] = \cos(\theta_1 + \theta_2) = \cos(\theta_1) \cos(\theta_2) - \sin(\theta_1) \sin(\theta_2).

Thus, we can express P P as a two variable function (in terms of θ 1 \theta_1 and θ 2 \theta_2 ). When P P is maximized, its partial derivatives are both equal to 0 , 0 , P θ 1 = 0 , P θ 2 = 0. \dfrac{\partial P}{\partial \theta_1} = 0 , \dfrac{\partial P}{\partial \theta_2} = 0 .

P θ 1 = 0 48 sin ( θ 1 + θ 2 ) 100 96 cos ( θ 1 + θ 2 ) + 42 sin θ 1 85 84 cos θ 1 = 0 (1) \dfrac{\partial P}{\partial \theta_1} = 0 \quad\implies \quad \dfrac{48 \sin(\theta_1 + \theta_2)}{\sqrt{100 - 96 \cos(\theta_1 + \theta_2)}} + \dfrac{42\sin\theta_1}{\sqrt{85 - 84\cos \theta_1}} = 0 \tag1

P θ 2 = 0 48 sin ( θ 1 + θ 2 ) 100 96 cos ( θ 1 + θ 2 ) + 56 sin θ 2 113 112 cos θ 2 = 0 (2) \dfrac{\partial P}{\partial \theta_2} = 0 \quad\implies \quad \dfrac{48 \sin(\theta_1 + \theta_2)}{\sqrt{100 - 96 \cos(\theta_1 + \theta_2)}} + \dfrac{56\sin\theta_2}{\sqrt{113 - 112\cos \theta_2}} = 0 \tag2

Because ( 1 ) = ( 2 ) (1) = (2) :

42 sin θ 1 85 84 cos θ 1 = 56 sin θ 2 113 112 cos θ 2 \dfrac{42\sin\theta_1}{\sqrt{85 - 84\cos \theta_1}} = \dfrac{56\sin\theta_2}{\sqrt{113 - 112\cos \theta_2}}

Square both sides of the equation, then utilize the identity sin 2 ( Z ) = 1 cos 2 ( Z ) , \sin^2 (Z) = 1 - \cos^2(Z) ,

9 a 2 [ 1 ( 85 a 2 84 ) 2 ] = 16 b 2 [ 1 ( 113 b 2 112 ) 2 ] \dfrac9{a^2} \left[1 - \left(\dfrac{85-a^2}{84} \right)^2 \right ] = \dfrac{16}{b^2} \left[1 - \left(\dfrac{113-b^2}{112} \right)^2 \right ]

Upon simplifying, a 2 + 169 a 2 + 56 = b 2 + 225 b 2 . (3) a^2 + \dfrac{169}{a^2} + 56 = b^2 + \dfrac{225}{b^2} \tag3.

Likewise we can also express P P as a two variable function (in terms of θ 1 \theta_1 and θ 3 \theta_3 ), and by using cos ( θ 2 ) = cos ( θ 1 + θ 3 ) = cos ( θ 1 ) cos ( θ 3 ) sin ( θ 1 ) sin ( θ 3 ) . \cos(\theta_2) = \cos(\theta_1 + \theta_3) = \cos(\theta_1) \cos(\theta_3) - \sin(\theta_1) \sin(\theta_3) .

Using the similar reasoning as above, we gather that

P θ 1 = 0 56 sin ( θ 1 + θ 3 ) 113 112 cos ( θ 1 + θ 3 ) + 42 sin θ 1 85 84 cos θ 1 = 0 (4) \dfrac{\partial P}{\partial \theta_1} = 0 \quad\implies \quad \dfrac{56\sin(\theta_1 + \theta_3)}{\sqrt{113 - 112 \cos(\theta_1 + \theta_3)}} + \dfrac{42\sin\theta_1}{\sqrt{85 - 84\cos \theta_1}} = 0 \tag4

P θ 3 = 0 56 sin ( θ 1 + θ 3 ) 113 112 cos ( θ 1 + θ 3 ) + 48 sin θ 3 100 96 cos θ 3 = 0 (5) \dfrac{\partial P}{\partial \theta_3} = 0 \quad\implies \quad \dfrac{56 \sin(\theta_1 + \theta_3)}{\sqrt{113 - 112 \cos(\theta_1 + \theta_3)}} + \dfrac{48\sin\theta_3}{\sqrt{100 - 96\cos \theta_3}} = 0 \tag5

Again, because ( 4 ) = ( 5 ) : (4) = (5):

42 sin θ 1 85 84 cos θ 1 = 48 sin θ 3 100 96 cos θ 3 49 a 2 [ 1 ( 85 a 2 84 ) 2 ] = 64 b 2 [ 1 ( 100 b 2 96 ) 2 ] \dfrac{42\sin\theta_1}{\sqrt{85 - 84\cos \theta_1}} = \dfrac{48\sin\theta_3}{\sqrt{100 - 96\cos \theta_3}} \quad \implies \quad \dfrac{49}{a^2} \left[1 - \left(\dfrac{85-a^2}{84} \right)^2 \right ] = \dfrac{64}{b^2} \left[1 - \left(\dfrac{100-b^2}{96} \right)^2 \right ]

Simplify once more a 2 + 169 a 2 + 30 = c 2 + 784 c 2 (6) a^2 + \dfrac{169}{a^2} + 30 = c^2 + \dfrac{784}{c^2} \tag6

Combining ( 3 ) (3) and ( 6 ) (6) yields

a 2 + 169 a 2 + 56 = b 2 + 225 b 2 = c 2 + 784 c 2 + 26. a^2 + \dfrac{169}{a^2} + 56 = b^2 + \dfrac{225}{b^2} = c^2 + \dfrac{784}{c^2} + 26 .

Hence, P = 169 = 13 , Q = 56 , R = 225 = 15 , S = 784 = 28 , T = 26. P = \sqrt{169} = 13, Q = 56, R = \sqrt{225} = 15, S = \sqrt{784} = 28, T = 26 . The answer is 13 + 56 + 15 + 28 + 26 = 138 . 13 + 56 + 15 + 28 + 26 = \boxed{138} .

0 pending reports

×

Problem Loading...

Note Loading...

Set Loading...